User avatar
 
ohthatpatrick
Thanks Received: 3808
Atticus Finch
Atticus Finch
 
Posts: 4661
Joined: April 01st, 2011
 
 
 

Re: Q22 - A recent survey indicates that

by ohthatpatrick Fri Dec 31, 1999 8:00 pm

Question Type:
Explain Discrepancy (EXCEPT)

Stimulus Breakdown:
Given that "survey says that avg # of books read per capita has gone down over last 3 years",
how can it be that "most bookstores have had increasing profits during the same period"?

Answer Anticipation:
Overall readership is down, but bookstore profits are up. Well, you don't have to READ a book to buy it. So maybe people are just buying books and not reading them. Maybe books are being sold at a higher price, so that even though volume is down, profits are higher. Since this is an EXCEPT, four wrong answers will help us reconcile these facts. One correct answer will either make the paradox all the more confusing or just do nothing to help explain it.

Correct Answer:
B

Answer Choice Analysis:
(A) Maybe this could show us a way that book reading, some of which is done from library resources, has gone down, even though it would have no impact on bookstores (or may even force people to buy contemporary fiction that they otherwise would have just checked out of the library).

(B) This is the answer! This does nothing. It actually says "bookstores are largely unaffected", so how would this help to explain a change in bookstore profits?

(C) Ah, so bookstores are making more profits from selling coffee, not books.

(D) Ah, so bookstores are profiting more on the individual sale of each book.

(E) Ah, so bookstores are making more profits from selling magazines, not books.

Takeaway/Pattern: These Explain EXCEPT questions can really get you turned around because the explanations can function in different ways. (A) makes it sound like bookstores ARE selling more books, since people can't get contemporary fiction at the library like they used to. (D) sounds like bookstores are selling the same number of books, but making more profit per book. (C) and (E) sound like bookstores are getting their increased profits from something ELSE they sell, besides books.

#officialexplanation
 
pele24
Thanks Received: 0
Forum Guests
 
Posts: 5
Joined: November 23rd, 2009
 
 
trophy
First Responder
 

Q22 - A recent survey indicates that

by pele24 Mon Jan 25, 2010 7:06 pm

Hi MLSAT Staff!
Could you, please, explain me why the answer choice B is right. I would rather choose A.
To my mind, B helps to resolve the discrepancy. It shows that antitheft equipment helped to increase profits.

Thank you!
User avatar
 
ManhattanPrepLSAT1
Thanks Received: 1909
Atticus Finch
Atticus Finch
 
Posts: 2851
Joined: October 07th, 2009
 
This post thanked 1 time.
 
 

re: Q22 - A recent survey indicates that

by ManhattanPrepLSAT1 Tue Jan 26, 2010 9:31 pm

Ok. So we're down to A and B. We need to keep the question stem in mind. The question stem says that "Each of the following, if true, helps to resolve the survey's apparently paradoxical results EXCEPT:"

What's important to keep in mind is that the answer choice needs to resolve the discrepancy between the two statements given in the stimulus. Answer choice B does seem to suggest that it was tightened security that allowed bookstores to increase their profits. But how does that reconcile the increased profits with lowered per capita reading? Answer choice B simply supports one of the two statements we're supposed to be reconciling. The correct answer must support how both statements can be true at the same time, not just one.

Answer choice A on the other hand suggests that people who would usually get books at the library (and not pay for them) are now forced to purchase them from bookstores. So, even if only half of the people who can no longer get these books at libraries actually turn up in bookstores to buy these books, we still have increased profits. And those who don't turn up in bookstores to buy these books, well they lead to the lowered per capita reading.

Hope this helps!
 
pele24
Thanks Received: 0
Forum Guests
 
Posts: 5
Joined: November 23rd, 2009
 
 
trophy
First Responder
 

Re: PT 53, S3, Q22 - A recent survey indicates that the average

by pele24 Wed Jan 27, 2010 11:38 am

Thank you! However, it is a tricky one, IMHO.
I could not understand how the choice A explained lowered per capita reading.
 
netkorea06
Thanks Received: 1
Forum Guests
 
Posts: 12
Joined: April 25th, 2010
 
 
 

Re: PT 53, S3, Q22 - A recent survey indicates that the average

by netkorea06 Wed Sep 22, 2010 2:35 pm

What about (c)? It shows that many bookstores can increase their profits. But how does that reconcile the increased profits with lowered per capita reading?
 
didi0504
Thanks Received: 0
Forum Guests
 
Posts: 23
Joined: October 20th, 2010
 
 
 

Re: PT 53, S3, Q22 - A recent survey indicates that the average

by didi0504 Thu Nov 25, 2010 8:25 pm

I have the same question as netkorea06
 
amanca
Thanks Received: 0
Forum Guests
 
Posts: 1
Joined: November 06th, 2010
 
 
 

Re: PT 53, S3, Q22 - A recent survey indicates that the average

by amanca Mon Nov 29, 2010 10:01 am

I'm going to go against Msherman in his explanation above of why B does not resolve the paradox. I think understanding B is definitely the key to this problem.

So, 4 of the 5 answers should give you a POSSIBLE reason why the bookstores could be experiencing increased profits in light of the decrease in books read. D and E people don't seem to be having a problem with understand, so I'll ignore those. First I'll explain C as two people asked above:

C: Reconciles the increased profits with less reading, by showing an alternative income stream for bookstore owners. In many ways, it's not much different from E, E shows that they could POSSIBLY be (remember, this is the key in resolving the paradox, you're showing a POSSIBLE resolution to the paradox, not an absolute resolution) receiving more income from increased magazine sales. In the same way, they could POSSIBLY be receiving more money by opening coffee bars.

Now, I think eliminating B is much more simple than msherman posted above. Here was my reasoning:

B: Answer B is showing not that antitheft equipment is a possible reason for increased profits, remember we're looking for a possible reason why profits were increased, BUT one possible reason why profits DID NOT DECREASE.

I hope that helps :)
User avatar
 
ManhattanPrepLSAT1
Thanks Received: 1909
Atticus Finch
Atticus Finch
 
Posts: 2851
Joined: October 07th, 2009
 
 
 

Re: PT 53, S3, Q22 - A recent survey indicates that the average

by ManhattanPrepLSAT1 Wed Dec 01, 2010 5:27 am

didi0504 Wrote:Now, I think eliminating B is much more simple than msherman posted above. Here was my reasoning:

B: Answer B is showing not that antitheft equipment is a possible reason for increased profits, remember we're looking for a possible reason why profits were increased, BUT one possible reason why profits DID NOT DECREASE.

Thanks for making that so much simpler... i completely agree. I forgot for a brief moment that people read these explanations after I answer the initial question. The point I was trying to make is that even if you were to concede the point that this is how bookstores got higher profits - better security or something - that you still couldn't use answer choice (B) to reconcile the two claims.

But fundamentally, I agree, this claim also fails to show how bookstores increased profits and instead shows why profits did not decrease.

amanca, your explanation for answer choices (C) and (E) were spot on as well.

Great catch!
 
apom
 
 
 
 

Re: PT 53, S3, Q22 - A recent survey indicates that the average

by apom Thu Feb 03, 2011 7:03 pm

B doesn't resolve the paradox at all because it is talking about a recent increase[b][/b] in shop lifting. The reasoning is based on a 3 year study.
 
mcrittell
Thanks Received: 5
Atticus Finch
Atticus Finch
 
Posts: 154
Joined: May 25th, 2011
 
 
 

Re: Q22 - A recent survey indicates that the average

by mcrittell Sat Sep 17, 2011 11:46 pm

But aren't we inferring too much in A if we assume that they--these random ppl that the AC doesn't specify--are purchasing books, thereby increasing profits?
User avatar
 
ManhattanPrepLSAT1
Thanks Received: 1909
Atticus Finch
Atticus Finch
 
Posts: 2851
Joined: October 07th, 2009
 
 
 

Re: Q22 - A recent survey indicates that the average

by ManhattanPrepLSAT1 Tue Sep 20, 2011 1:32 pm

mcrittell Wrote:But aren't we inferring too much in A if we assume that they--these random ppl that the AC doesn't specify--are purchasing books, thereby increasing profits?

Remember, this is not an inference question or an assumption question. All we need to do is find an answer choice that would help to resolve the paradox, and answer choice (A) would offer a good reason why folks would be out there purchasing books - they can't get those books in the library any more.

On Explain/Resolve questions you don't actually need to prove how something happened, you just need to offer a possibility that would help clear things up. This is similar to Strengthen/Weaken questions where you do not need to prove/disprove the conclusion, you just need to nudge it in the right direction.

Does that answer your question?
 
shingctse
Thanks Received: 0
Forum Guests
 
Posts: 3
Joined: October 25th, 2011
 
 
 

Re: Q22 - A recent survey indicates that the average

by shingctse Mon Apr 30, 2012 3:07 pm

My understanding of why answer choice B.) is wrong is a bit different than the approach demonstrated previously.

The seeming paradox is that books read annually per capita declined in the previous three years, but profits in bookstores increased in the same period.

Answer choice b.) indicates that the anti-theft equipment helped prevent shoplifting that hit recently, but wouldn't be able to account for the other two years.

Did I do that correctly? It just seems that had they left out the portion about three years, one could reasonably argue that their profits went up because people were no longer stealing all of their books, but since their profits have increased in the last three years, the recent change in security cannot account for the entire duration of increased profits.
 
timmydoeslsat
Thanks Received: 887
Atticus Finch
Atticus Finch
 
Posts: 1136
Joined: June 20th, 2011
 
 
trophy
Most Thanked
trophy
First Responder
 

Re: Q22 - A recent survey indicates that

by timmydoeslsat Mon Apr 30, 2012 5:19 pm

I would not dismiss it for that reason alone. Recently could account for the past 3 years. I would not bring assumptions along as to what qualifies for recent. Why is only the last year considered recent?

With answer choice B, its problem is that it does not tell us that these bookstores were being hit by shoplifters before the equipment took effect. I would suppose that if we did know that they were being hit by shoplifters prior to the installation, we could have this help us with increased profits.
 
jamiejames
Thanks Received: 3
Atticus Finch
Atticus Finch
 
Posts: 116
Joined: September 17th, 2011
 
 
 

Re: Q22 - Brain-scanning technology provides

by jamiejames Wed May 09, 2012 3:16 pm

How does C reconcile the gap between decreased reading and increased sale. Does opening a coffee bar mean that people will read less? I understand it will increase profits, but it won't explain why there was a also a decline in reading.
 
timmydoeslsat
Thanks Received: 887
Atticus Finch
Atticus Finch
 
Posts: 1136
Joined: June 20th, 2011
 
 
trophy
Most Thanked
trophy
First Responder
 

Re: Q22 - A recent survey indicates that

by timmydoeslsat Thu May 10, 2012 3:18 pm

jeastman Wrote:How does C reconcile the gap between decreased reading and increased sale. Does opening a coffee bar mean that people will read less? I understand it will increase profits, but it won't explain why there was a also a decline in reading.

It helps for the two situations to coexist. People can read less and these bookstores can increase profits. It shows that the bookstores do not have to rely as heavily on the book sales. So while we would expect the bookstores to have lost revenue from people not reading as much, this coffee sale endeavor is what is reponsible for the increased profits.

The discrepancy is how can these bookstores have their profits go up while the average # of books read per person has gone down?

I also noticed that the post topic needs to be renamed.
 
alexg89
Thanks Received: 9
Jackie Chiles
Jackie Chiles
 
Posts: 39
Joined: July 24th, 2012
 
 
 

Re: Q22 - A recent survey indicates that

by alexg89 Fri Aug 31, 2012 5:17 pm

Must allow for decrease in per capita books read over last three years and it must account for increase in bookstore profits over the last three years

Key: Must allow both sides to be factually correct and either show a possible cause of the paradox, or add a statement which allows them to both coexist. Does not have to prove something without a doubt, just helps resolve it.

A: If libraries don’t carry as many popular novels then this makes it plausible that some people will not read them. Hence, it explains why there could be a decline in the per capita books read. Now conversely, it is plausible that some people that have read those novels from the library will now purchase them from bookstores. Also note that it says recent in answer choices A and B. This is an attempt to lure you. Recent is used ambiguously in A while it is placed into much more specific context in B.

B: Superficially this appears to explain the increase in profits from the bookstore. It can be argued that it can peacefully coincide with a decrease in books read per capita. However, it says the anti-theft equipment is NEW, by definition meaning that it did not exist before. This places its use of recent in a more specific context. The answer is thus correct because it does not correlate with the correct span of time.

C: Accounts for the increase in bookstore profits while simultaneously allowing the decrease in books read per capita to still coincide.

D: Explains why bookstores would make more profit. Additionally, you could plausibly accept a scenario where people buy less and read less books because they are tending to buy more expensive ones.

E: Easily explains why bookstores would make more profit. Also, it helps gives a reason to help explain why people would not read as many books. Namely, a lack of free time.
 
shirando21
Thanks Received: 16
Atticus Finch
Atticus Finch
 
Posts: 280
Joined: July 18th, 2012
 
 
 

Re: Q22 - A recent survey indicates that the average

by shirando21 Wed Oct 24, 2012 2:28 pm

mattsherman Wrote:
mcrittell Wrote:But aren't we inferring too much in A if we assume that they--these random ppl that the AC doesn't specify--are purchasing books, thereby increasing profits?

Remember, this is not an inference question or an assumption question. All we need to do is find an answer choice that would help to resolve the paradox, and answer choice (A) would offer a good reason why folks would be out there purchasing books - they can't get those books in the library any more.

On Explain/Resolve questions you don't actually need to prove how something happened, you just need to offer a possibility that would help clear things up. This is similar to Strengthen/Weaken questions where you do not need to prove/disprove the conclusion, you just need to nudge it in the right direction.

Does that answer your question?


We need to make further assumption from what A tells us. (the part that people will instead go to a bookstore to purchase books) You mean in a correct answer of Resolve Paradox questions, it is allowed to make further assumptions from what the choice simply says?

Is resolve a paradox the only question type that making assumptions from what the choices tells is allowed?
 
timsportschuetz
Thanks Received: 46
Elle Woods
Elle Woods
 
Posts: 95
Joined: June 30th, 2013
 
 
trophy
First Responder
 

Re: Q22 - A recent survey indicates that

by timsportschuetz Wed Nov 13, 2013 12:38 am

To be honest, I have read all of the above explanations attempting to explain why answer choice (B) is correct, however, none of them hit the nail on the head. Firstly, let me dispute some of the above comments: the timespan is NOT a valid reason for choosing this answer choice! We are not told when exactly the anti-theft devices were installed. Furthermore, we are not told the definition of "recent" - "recent" could mean 3 years ago, it could mean 2 weeks ago, or it could mean 15 years ago. "Recent" is a very tricky word! Unless "recent" is specifically defined, we cannot conclude "recent" to mean one time-span over another. Thus, "recent" should provide you with SOME assurance that this answer choice may be credited, however, you must dig deeper to find the main reason why answer choice (B) is correct.

Notice how answer choice (B) compares bookstores to other retail businesses. All we can validly conclude from this answer choice is that bookstores MAY have realized increased and/or higher profits compared to other retailers! However, this is completely irrelevant to the stimulus! We are trying to explain the unexpected result of the paradox. Stating that bookstores had potentially higher profits than other retailers has ZERO impact on resolving the paradox between the lower average numbers of books read annually per capita and the increased profits of bookstores during this same period.
 
kyuya
Thanks Received: 25
Elle Woods
Elle Woods
 
Posts: 77
Joined: May 21st, 2015
 
This post thanked 3 times.
 
 

Re: Q22 - A recent survey indicates that

by kyuya Thu Jun 11, 2015 7:14 pm

The amount of books being read is down (so people should be buying less books, right?), but book stores are earning more profits. How is this possible when they make their money from books?

1.) Perhaps they found a new way of making money besides books
2.) A new grant from the government?

Essentially, is there some other form of income then? I think a few of the answers address these potential issues.

I'll get rid of the more obvious ones first.

(C) It goes along with some of the alternatives that I suggested in my numbered post above. So they now have an alternative way to get money -- they don't necessarily need to selling books. Therefore, if book sales are down, the decrease is perhaps compensated by the amount of coffee they are selling.

(D) Suggests the books they are now selling are more expensive. So even if they are selling less books as it states in the stimulus, it doesn't matter because the books are now more expensive. Perhaps selling 1 hardback book is the equivalent to selling 2 paper back books (in terms of $), so even if they sold a little over half of the amount of books that they did in the previous year, the greater monetary value would be more than enough to make profit for books stores rise.

(E) Suggests that book stores are getting customers that they never had before, which can help supplement the money lost. They are purchasing magazines which gives the book store money, while at the same is consistent with the amount of books sold per capita falling.

Okay onto what looks like the two more controversial answers, (A) and (B).

(A) In similar fashion to (E), (A) suggests that book stores may not be getting customers they previously did not have access to before. Since public libraries cannot provide a certain subset of readers books in the same way some of the book stores can, these customers now come to the book stores which raises their income, while at the same time not necessarily meaning that MORE books are being read, but simply that a shift of where a subset of people are getting their literature has changed, to the advantage of book stores. In short, its consistent with the idea of book sales going down, but book stores getting more money.

(B) However, doesn't explain why their money is going up. Basically, book stores are not being shoplifted from, while other places are. Does the fact that book stores have remained untouched mean that they will now necessarily gain money from this? Of course not, they will stay the exact same. It does suggest however, that maybe they have increased profits RELATIVE to other places which is perhaps where some of the confusion on the questions stems from.

I think a (A) is tricky because if you do not catch the fact that "public libraries" and "book stores" are not the same thing, it becomes difficult to know what means what. When a group is explicitly mentioned in LR stimuli its always a good idea to keep that group in your head distinctly.
 
matthughes2
Thanks Received: 0
Vinny Gambini
Vinny Gambini
 
Posts: 8
Joined: November 18th, 2015
 
 
 

Re: re: Q22 - A recent survey indicates that

by matthughes2 Wed Nov 25, 2015 3:23 pm

mattsherman Wrote:Answer choice A on the other hand suggests that people who would usually get books at the library (and not pay for them) are now forced to purchase them from bookstores. So, even if only half of the people who can no longer get these books at libraries actually turn up in bookstores to buy these books, we still have increased profits. And those who don't turn up in bookstores to buy these books, well they lead to the lowered per capita reading.


Doesn't this make it equally likely that ALL of the people who can't get Dan Brown at the library are going to the bookstore for it, and therefore reading can be at least equal while profits are up, therefore not explaining paradox?

It seems like the same tendency to infer that while some are buying (and therefore P^) that any number of the rest are simply not reading OR buying and therefore Rv could logically lead you in the other direction.

I guess this solves the paradox IF what you've said is true, and is the ONLY choice that does so and is therefore correct, but I don't know why one would or should make that leap.

I mean, for what it's worth, the rest of the choices CLEARLY help to resolve the paradox so one could get this right with PoE. I just have a hard time bringing something in out of left field in order to make an answer correct.